Comparing 5^51 and 2^120

Поделиться
HTML-код
  • Опубликовано: 7 июл 2021
  • Join this channel to get access to perks:→ bit.ly/3cBgfR1
    My merch → teespring.com/stores/sybermat...
    Follow me → / sybermath
    Subscribe → ruclips.net/user/SyberMath?sub...
    Suggest → forms.gle/A5bGhTyZqYw937W58
    If you need to post a picture of your solution or idea:
    intent/tweet?text...
    #ChallengingMathProblems #ComparingNumbers
    EXPLORE 😎:
    Solving a Quick and Easy Functional Equation: • Solving a Quick and Ea...
    Solving a Polynomial System in Two Ways: • Solving a Polynomial S...
    A Trigonometry Challenge: • A Trigonometry Challenge
    PLAYLISTS 🎵 :
    Number Theory Problems: • Number Theory Problems
    Challenging Math Problems: • Challenging Math Problems
    Trigonometry Problems: • Trigonometry Problems
    Diophantine Equations and Systems: • Diophantine Equations ...
    Calculus: • Calculus

Комментарии • 291

  • @1Rom4ik
    @1Rom4ik 3 года назад +200

    My suggestion is to present 5^51 as 125^17, and 2^120 as 128^(120/7) which is 128^17.1.. . Since 128>125 and 17.1...>17 the second number is of course bigger. p.s your solution is very close, I solved it before watching)

    • @spirome28
      @spirome28 3 года назад +2

      me too

    • @satyapalsingh4429
      @satyapalsingh4429 3 года назад +1

      Me too

    • @Sam_on_YouTube
      @Sam_on_YouTube 3 года назад +1

      Likewise. Solved it in my head. Thougg I screwed up a little. I thought 128 was 2^6. Oops.

    • @pietergeerkens6324
      @pietergeerkens6324 3 года назад

      Yep. Almost could do this in my head this way; but not quite.

    • @mmh1922
      @mmh1922 3 года назад

      Nice!

  • @5gonza541
    @5gonza541 3 года назад +49

    Every time I see this kinda questions I always pick the higher exponent, it happens to work too often lol

    • @SyberMath
      @SyberMath  3 года назад +3

      Haha! That's cool!

    • @duckymomo7935
      @duckymomo7935 2 года назад

      It has to be sufficiently large enough

    • @namraksubba5833
      @namraksubba5833 2 года назад +1

      Yea! This is kind of a rule, for two numbers say 'a' and 'b' more than 2, the rule applies where the lower value number with higher value power is always the greater than the higher value number with lower value power, where numbers are given in the form "a^b"and "b^a ".
      Note: a and b are to be same.i.e. either the base is to be same or power to be same for this rule to apply so in this case we gotta make power same or base same .

  • @georgesbv1
    @georgesbv1 3 года назад +22

    I multiplied both sides with 2^51. And applied compaction (10^3)^17 vs 2*(2^10)^17

    • @kuokenwei607
      @kuokenwei607 3 года назад +2

      Do the same way😎

    • @malcom91
      @malcom91 3 года назад

      This is the way I solved it too

    • @aaykat6078
      @aaykat6078 2 года назад

      This one is definitely smarter and quicker

  • @nouration9685
    @nouration9685 3 года назад +13

    I solved using the fact that 2^10 is 1024, which is a little over 10^3 (it's a very important fact if you work with binary numbers a lot)
    we got 5^51 and 2^120, if we multiply both by 2^51
    so we got 10^51 on one side [(2*5)^51] and 2^171 on the other
    since we know 2^10 > 10^3, then we can raise both sides to the power of 17, and we get
    2^170 > 10^51
    and we need to compare 10^51 to 2^171
    2^171 > 2^170 > 10^51

    • @davidbelk46
      @davidbelk46 3 года назад +2

      That was the first thing I thought of as well.

  • @samarthsuthar3832
    @samarthsuthar3832 2 года назад +26

    It can be really simple by just taking log to the base e on both the sides...
    On lhs we'll have
    51(ln 5)
    And rhs we'll have
    120(ln2)
    We'll eventually get lhs as
    82.11 and
    Rhs as 83.16.
    Clearly rhs>lhs
    Hence 2²¹⁰>5⁵¹...

    • @alimuhammadnasir1571
      @alimuhammadnasir1571 2 года назад +9

      I think the fact that he went to this lenght was to try and avoid using a calculator

    • @samarthsuthar3832
      @samarthsuthar3832 2 года назад +1

      @@alimuhammadnasir1571 Oh yes.. it can be possible...👍

    • @samarthsuthar3832
      @samarthsuthar3832 2 года назад +2

      @@harshalkashyap8367 Bhai Bhai...🔥😂😂

    • @baydamm
      @baydamm 2 года назад

      Not true

  • @echandler
    @echandler 3 года назад

    Congrats on reaching 25K subs. Again, a very nice video.
    For those of you young enough to consider math competitions, you may need to make this type of argument. (I only did one of these, a state competition, back in 1970!)

  • @zxulokulinana
    @zxulokulinana 3 года назад +1

    Interesting ~ so important to see the detail

  • @emanuellandeholm5657
    @emanuellandeholm5657 3 года назад +5

    I did it in my head. :) Tried the 5^50 first, saw that the inequality was going in the wrong direction. Then I realized 119 is a multiple of 17, so I went with 2^120 > 2^119 and this time the inequality came out right, (2^7)^17 > (5^3)^17

  • @rssl5500
    @rssl5500 3 года назад

    Nice solution from you thank you and it’s very interesting that in can be solved in a number of ways (some of which have been pointed out by other viewers)

  • @pierreabbat6157
    @pierreabbat6157 3 года назад +4

    First thing I thought of was to take the cube root of both sides; this gives 5^17?2^40. Then factor 5^17 into (5^3)^5×5^2 and 2^40 into (2^7)^5×2^5. This gives 125^5×25

  • @T0NYD1CK
    @T0NYD1CK 3 года назад +4

    If you can remember that log10(5) is 0.69897 and log10(2) is 0.30103 you are almost there.
    Using 0.7 and 0.3 as the values gives the log10 of the answers as: 357 and 360.
    As I rounded up the former and rounded down the latter that must indicate the answer.

  • @pedroloures3310
    @pedroloures3310 3 года назад +1

    I liked your solution, it taught me alot! Thanks for the video!

  • @pdean6140
    @pdean6140 3 года назад +1

    Very good explanation ! Thank you very much ! Video is useful for novice teachers .

    • @SyberMath
      @SyberMath  3 года назад

      Glad it was helpful!

  • @RisetotheEquation
    @RisetotheEquation 3 года назад +2

    Awesome, Syber! Next up is e^pi vs pi^e, right? I plan to do a video on that sometime soon myself, so it will be interesting to see which approach we take :)

    • @SyberMath
      @SyberMath  3 года назад +1

      Sounds good!

    • @fxhw7754
      @fxhw7754 2 года назад +1

      RPBP did this, a general rule of thumb for x^y>pi^e

  • @ryanxu1718
    @ryanxu1718 3 года назад

    I really enjoy ur way of thinking… and btw, I just use intuitive guess to get right this time… because this is why people want more “power” than a bigger but not big enough “base”. ;-)

  • @PunmasterSTP
    @PunmasterSTP 3 года назад

    Thanks for another awesome video! Personally the first thing that popped into my mind was to rewrite 5 as 2^(log of 5 base-2) so we get (2^(log5))^51 and it turns into a comparison of 51*log(5) and 120. But I'm seeing a lot of different methods, and I think some are better, and definitely don't require a calculator...

  • @242math
    @242math 3 года назад

    understand the steps perfectly bro, thanks for sharing

  • @RisetotheEquation
    @RisetotheEquation 3 года назад +2

    Very nice solution! You can also take the cube root of both to get 5^17 vs 2^40. Next notice that 2^10 = 1024, but just call it 1000 = 10^3. Now compare 5^17 vs 10^12. Divide both sides by 5^12 and compare 5^5 to 2^12. 3125 < 4096 so 2^120 is bigger.

  • @tonyhaddad1394
    @tonyhaddad1394 3 года назад +1

    Awesome problem !!! i like these types 😍

  • @wiseSYW
    @wiseSYW 3 года назад +1

    your first method works fine. 10th root of 5 is pretty small, close to 1, you could assume multiplying it only increases by 10%, while 2^12 is still ~33% larger than 5^5

  • @math_person
    @math_person 3 года назад +1

    I did it this way:
    If (5 ^ 51) / (2 ^ 120) < 1 or > 1, then we know which is greater.
    = ((5/4) ^ 51 ) * (1 / (2^18))
    = ((1.25) ^ 50) * (1.25) / (2 ^ 18)
    = ((1.5625) ^ 25) * (1.25) / (2 ^ 18)
    = (1.5625/2)^18 * (1.25^8)
    = ((0.78125)^18) * ((1.25)^8)
    = ((0.78125 * 1.25) ^ 8) * ((0.78125)^10)
    = ((0.9765625)^8) * ((0.78125)^10)
    As both the terms are less than 1, it means that product of their exponentials is also less than 1. QED

  • @fmakofmako
    @fmakofmako 3 года назад +17

    My attempt without watching/reading comments: is that I want a close inequality to try to bound this. 5^3 = 125 < 128 = 2^7, this is the smallest power that got a reasonably tight bound. Actually just for fun let's try a loose bound: 5^1 < 2^3, then we get 5^51 < 2^153, which is way too loose. Next 5^2 < 2^5, so 5*25^25 < 5*2^125 < 2^128, which is much better already.
    There is a clue built into the question since 51 is a power of 3 and 120 is a power of 8, but that could be misleading. Anyways, 125^17 < 2^(7*17) = 2^119 < 2^120.
    Edit: it would be difficult to get a tighter bound because 5^51 < 2^118.42

    • @tastypie2276
      @tastypie2276 3 года назад +2

      I did the same approach. It is easier to compare small numbers. And then you just need to increase the powers: 5^3 < 2^7 => (5^3)^17 < (2^7)^17 => 5^51 < 2^119 => 5^51 < 2^120 => DONE!

    • @yogamulyadi9183
      @yogamulyadi9183 2 года назад

  • @billcame6991
    @billcame6991 2 года назад

    I took the cube root of both sides to produce 5^17 and 2^40. These were "smaller" numbers so it made comparing a little easier.
    3,125^3 x 25 vs 1,024^4.
    1,042^3 x 675 vs 1,024^4.
    If you take log base 2 of each, you get (estimates)
    30.3 (generous) + 9.3 (guestimate) vs 40 (exact)

  • @dontsetyourlimitsyt4939
    @dontsetyourlimitsyt4939 3 года назад +2

    Alternate method:
    Take log on both sides. Compute its value by taking log 2~0.3010 and log 5~0.6989. Find the the number with larger value and you are done.
    log5⁵¹=35.6439 & log2¹²⁰=36.120

    • @justabunga1
      @justabunga1 3 года назад +2

      That would be true, but most people don't know what the value of log(2) and log(5) is, so we have to leave that as an exact answer since no calculators are allowed. However, we can substitute both of the log values for something else. We know that log(2)+log(5)=log(10)=1. Therefore, we can let one of the log values by comparing the other value. I'll try log(2)=1-log(5). We can tell that log(2)51log(5). Therefore, 5^51

    • @albertmcchan
      @albertmcchan 3 года назад +1

      2^10 = 1024 ≈ 10^3, → log10(2) slightly above 0.3
      log10(5) = log10(10/2), slightly below 1 - 0.3 = 0.7
      log10(2^120) > 120 * 0.3 = 36.0
      log10(5^51) < 51* 0.7 = 35.7 → 2^120 > 5^51

    • @justabunga1
      @justabunga1 3 года назад

      @@albertmcchan I like your idea, but we have to remember that both log(2) and log(5) have an approximate answer, but since calculators aren't allowed and can't use in our own heads, we can just leave those as an exact answer. What you can tell from there is that 10 is the LCM of both 2 and 5 in order to make everything the same with the comparison of both values to make things a lot of easier since log(2)+log(5)=log(10)=1. That's why we have to substitute in one of those values in 120log(2) and 51log(5), and then go from there.

  • @zafarmahmood8992
    @zafarmahmood8992 3 года назад

    It helpe us reckon the long looking calculation instead shortcut to answer

  • @haricharanbalasundaram3124
    @haricharanbalasundaram3124 3 года назад +2

    Great video as usual... could you make a video on Rolle's theorem, Lagrange's mean value theorem, and it's application in solving equations and such? I know what the statement says, but I'm unable to apply it, so any help is much appreciated. Thanks in advance :D

  • @misternegative2759
    @misternegative2759 2 года назад

    i did binomial distribution , 5^51 can be written as (4+1)^51 and in case of 2^120 , 4^60, since there is 1 is case of (4+1) it becomes easier and hence we can clearly see which one is bigger , even if the nature of power(exponents) if you see in graph the curve is high so obv 2^120 is higher if u guessing

  • @user-zz3hk4rq9g
    @user-zz3hk4rq9g 3 года назад +4

    Find value of 51log5 and 128log2
    Then the logarithm value of second number will be larger
    So second number will be larger

    • @vivianpaul2078
      @vivianpaul2078 3 года назад +1

      Exactly xd

    • @rishabhsetty3109
      @rishabhsetty3109 3 года назад

      I think the point is to not use a calculator, unless u know log2 and log5 off the top of ur head

    • @user-zz3hk4rq9g
      @user-zz3hk4rq9g 3 года назад

      @@rishabhsetty3109 hmmm
      Ok i Know the values of log2,3,5,7
      It helps me

    • @Sarif_boy_amit_
      @Sarif_boy_amit_ 2 года назад +1

      Yeah

  • @jaggisarma9513
    @jaggisarma9513 3 года назад +1

    Sir,using logarithms ,we find that2^120 > 5^51as mantissa of 120x0.3010 > mantissa of 51x.6998(log5)

  • @3r3nite98
    @3r3nite98 3 года назад

    This video is Greater than both values lol.
    Also Im glad adway kumar commented too.

  • @raivogrunbaum4801
    @raivogrunbaum4801 3 года назад +5

    5^51=125^17

  • @shanehebert396
    @shanehebert396 3 года назад

    Find log base 2 of 5, multiply that by 51. compare that result with 120. Alternately, (ln 5) * 51 compared to (ln 2) * 120, whichever is larger.

  • @septembrinol1
    @septembrinol1 8 месяцев назад

    I loved this method!

    • @SyberMath
      @SyberMath  8 месяцев назад

      Glad you liked it!

  • @moeberry8226
    @moeberry8226 3 года назад +1

    Rewrite 5 as 10/2 and then take log base 10 on both sides and you will obtain 51(log10-log(2))= 51(1-log(2)), assuming we don’t know the value of log(2) we go back to the definition of the logarithm. Log(2) means what power must base 10 be raised to obtain 2 or another words 10^y=2. Note that 3/10

    • @justabunga1
      @justabunga1 3 года назад +1

      exactly :) I like your idea

    • @andrewhone3346
      @andrewhone3346 3 года назад

      Or without logs, note that 2^10 is just a bit bigger than 10^3, so writing 5=10/2 gives the ratio 2^120/5^51=2*(2^170/10^51)>2

  • @nicogehren6566
    @nicogehren6566 3 года назад

    nice trick sir thanks

  • @hakjaolai
    @hakjaolai 3 года назад

    The principle used to solve this problem is essentially the same as the one used to solve the problem "Comparing 31^11 and 17^14" in another video of yours. Both are based on showing the upper bound of one number is less than the exact value or the lower bound of another. The difference is how to do the bounding and this is greatly facilitated by the "nice relationship" between the numbers in the problems. For this video, it would be interesting to see how one can solve the problem without using a calculator if the second number is 2^118 instead of 2^120

  • @vishalmishra3046
    @vishalmishra3046 3 года назад +1

    *5^51 = 125^17 < 128^17 = 2^119 < 2^120, hence 5^51 < 2^120*

  • @rahmatmatematika6534
    @rahmatmatematika6534 2 года назад

    Thank you from a math teacher in indonesia

  • @Kevsunsix
    @Kevsunsix Год назад

    Multiply by 2^51 on both sides. The reason for this is to convert 5^51 to a number of order 10. It becomes much easier to deal with.
    We are now comparing:
    10^51 vs 2^171.
    Here’s the thing, I know that 2^10 is 1024, thus it is greater than 10^3.
    What’s so significant about this? Well, I know that (10^3)^17 = 10^51 but I also know that (2^10)^17 = 2^170 which is less than 2^171. But since 2^10 > 10^3 (as we’ve established) that must mean that 2^170 > 10^51. Thus meaning that 2^171 > 10^51 as well.
    As such 2^120 > 5^51, which is the final answer.

  • @eduardomalacarne9024
    @eduardomalacarne9024 3 года назад

    Good question of this type, its incredible that's 2^120/5^51≈3

  • @dibyojyotibhattacharjee5349
    @dibyojyotibhattacharjee5349 3 года назад

    seriously finally this one....

  • @oenrn
    @oenrn 3 года назад +2

    5⁵¹ = (5³)¹⁷ = 125¹⁷
    2¹²⁰ = 2 × 2¹¹⁹ = 2 × (2⁷)¹⁷ = 2 × 128¹⁷
    Clearly the second is larger.

  • @ethanbottomley-mason8447
    @ethanbottomley-mason8447 3 года назад

    You can solve using log_2 pretty easily. log_2(5^51) = 51log_2(4*5/4) = 102 + 51log_2(5/4). The rhs becomes simply 120. Now we have to figure out if 102 + 51log_2(5/4) < 120 => log_2(5/4) < 18/51. This is trhe since 18/51 > 1/3 > log_2(5/4). It is easy to see that log_2(5/4) < 1/3 since (5/4)^3 = 125/64 < 2.

  • @grammarnazi8877
    @grammarnazi8877 2 года назад +1

    6:34 my life changed all of a sudden 😂

  • @chamsderreche5750
    @chamsderreche5750 2 года назад

    I had a different approach for this one, I took 8 as a common factor and compared 2^14^8 with 5^6^8, 2^14=16384>15625=5^6 so 5^48 < 2^112, and we have that 2^8=256>125=5^3, so 5^51 < 2^120

  • @addictedroaster1573
    @addictedroaster1573 2 года назад

    This just means justice to integers.... Doesn't matter if it's in power or not

  • @leejimmy90
    @leejimmy90 2 года назад

    Nice vid! Please do a comparison between 5^52 and 2^120

    • @SyberMath
      @SyberMath  2 года назад

      Thank you!

    • @italixgaming915
      @italixgaming915 2 года назад

      First of all since in R+, x^4>y^4 x>y we just need to compare 5^13 and 2^30.
      2^30/5^13=2^43/10^13
      So the problem is equivalent to: is 2^43 bigger than 10^13?
      2^43=(2^10)^4x2^3=(1024)^4x8.
      We already know that 2^43>1000^4x8=8x10^12 but this is not enough. We need to estimate 1024^4 more precisely.
      The problem can be rewritten: is 1024^4 bigger than 1.25x10^12?
      We are not going to calculate in detail: 1024^4

  • @leecherlarry
    @leecherlarry 3 года назад +6

    compi outputs a True for this hehe:
    *5^51 < 2^120*

    • @keescanalfp5143
      @keescanalfp5143 3 года назад +2

      I can see your statement, hehe.
      I can't see how you did

    • @leecherlarry
      @leecherlarry 3 года назад +2

      @@keescanalfp5143 i never do. i have compi for this. 😁

  • @advaykumar9726
    @advaykumar9726 3 года назад +7

    2^120 is bigger
    Take log of both expressions and sub value of log 2 and log 5 and you will see 120 log 2> 51 log 5 so 2^120 is bigger

    • @sujanshankarbhowmick9381
      @sujanshankarbhowmick9381 3 года назад

      For the summation 2+3^3+6+7^7+10+11^11+.............+(4x+2)+(4x+3)^(4x+3)=z^n, where n>1, how many +ve integer solutions for z are possible? Can you solve this

    • @georgesbv1
      @georgesbv1 3 года назад +1

      doesn't make sense. Maybe if you pull up lg 5 = 1 - lg2. And try to prove (120 + 51) lg2 > 51

    • @MichaelJamesActually
      @MichaelJamesActually 3 года назад +1

      Tough to do without a calculator. At least for me.

    • @srijanbhowmick9570
      @srijanbhowmick9570 3 года назад

      @@sujanshankarbhowmick9381 I think the answer is 0

    • @snoopywlk
      @snoopywlk 3 года назад +1

      I used this method too. log 2 is about 0.3 and log 5 is about 0.7. So 120 log 2 is about 36, and 51 log 5 is about 35.7. So 2^120 is larger.

  • @jorgelenny47
    @jorgelenny47 3 года назад

    I went for 5^51 = 5*(5^5)^10 = 5*3125*10 vs (2^12)10 = 4096^10. Taking the tenth root on both sides and dividing by 3125, you get 4096/3125 = 1.310 vs 10rt(5). Finally, you can raise both sides to the 10th power and check if 1.31^10 is bigger than 5, which it is (in fact, 1.31^6 is already bigger) so 2^120 is bigger

  • @sajidmushfique6375
    @sajidmushfique6375 2 года назад

    That's math with common sense . Awesome job 👌

  • @vishalmishra3046
    @vishalmishra3046 3 года назад

    *Strategy* Get base close enough to each other by factoring the exponents -
    Since 51 = 3x17 and 5^3=125, therefore LHS = 5^51 = 125^17.
    Since 2^7 = 128 is close but greater than 125, therefore RHS = 2^120 = 128^(120/7) = 128^(17+1/7)
    Since both base (125 < 128) and exponent (17 < 17+1/7) are smaller, therefore LHS < RHS, hence *5^51 < 2^120*

  • @Josh-cd3zf
    @Josh-cd3zf 3 года назад

    My method would be to rewrite this as comparing 51*log base 2 of 5 with 120. Notice that 125 < 128, meaning that log base 2 of 5 < 7/3, so the left hand side is less than 119, implying that 5^51 < 2^119. Since 2^119 < 2^120, 5^51 < 2^120

  • @iamtrash288
    @iamtrash288 3 года назад

    My way is very heuristic. Take logs, order is preserved. We have
    51ln5 v 120ln2
    17ln5 v 40ln2=20ln4
    ln 4 ~ ln5 (very small difference too lazy to evaluate through series)
    Meaning rhs is greater

  • @coolmangame4141
    @coolmangame4141 3 года назад

    I looked at their digit count using formula Floor(log_10(n)) + 1 and found out 5^51 has 36 digits, 2^120 has 37 digits. So 2^120 > 5^51, although their count are almost equal so this was a close call

  • @sankarrao1100
    @sankarrao1100 3 года назад +1

    Taking log, and observing log function that it changes very low for a change in input, i.e log 2~~log 5, which makes 120 >>51, therefore 2^120 is greatest

    • @tomkerruish2982
      @tomkerruish2982 3 года назад

      But log 5 > log 4 = 2 log 2; your approximation is a bit too loose. In base 10, log 2 is close to 0.3 and log 5 is close to 0.7. 51 log 5 is about 35.7 while 120 log 2 is about 36.
      As others have pointed out (I've only looked at comments, haven't yet watched the video), 5^3 < 2^7, so 5^51 = (5^3)^17 < (2^7)^17 = 2^119 < 2^120.

    • @justabunga1
      @justabunga1 3 года назад +1

      @@tomkerruish2982 let's not carry out what the approximation of log(2), log(3), log(5), etc. just because you can think in your head, but we have to remember the log of something that is not powers of 10 is not an exact answer but an approximate answer. We can only leave that as log(2) and log(5) as an exact answer. However, both log(2) and log(5) have something in common, which is to take the LCM of both 2 and 5. That's 10. Using what we know from the properties of logarithms, log(2)+log(5)=log(10)=1. We can substitute one of the values in to make the comparison of another value to see which is bigger. You can say that 120log(2)=120(1-log(5))=120-120log(5). From there, we know that 120log(5)>51log(5). When you multiply a negative number, the answer is a negative number. Then, adding 120 to -120log(5) yields a positive number, so 120log(2)>51log(5). Therefore, this yields 2^120>5^51.

  • @manoharkanade7383
    @manoharkanade7383 2 года назад

    We can divide both sides by 3125 ^10 so we get 1

  • @golddddus
    @golddddus 3 года назад +3

    log(2) ≈ 0.3, then log(5)=0.7 120 log (2) vs. 51 log (5), 360 > 357 😍

    • @davidjames1684
      @davidjames1684 3 года назад +1

      Try it with 5^59 vs. 2^137. 59*0.7 = 41.3. 137*0.3 = 41.1. That would imply that 5^59 > 2^137 but it is not. Your log approximations have enough error to not work when the 2 expressions are almost equal. This is cuz Log(2) ≈ .301 and Log(5) ≈ .699. 59*0.699 = 41.241 and 137*0.301 = 41.237 so even going to a 3rd decimal digit is not enough. 59*log(5) = 41.239... and 137*log(2) = 41.241... so now we can see 2^137 is larger than 5^59, but just barely. Also, in your solution, I think you mean 36 > 35.7.

    • @justabunga1
      @justabunga1 3 года назад

      We have to remember that log(2) and log(5) are irrational numbers. Therefore, you can only leave those as an exact answer. However, you can see that both log(2) and log(5) have something in common by taking the LCM of those values that is inside the log of an argument, which is 10. log(2)+log(5)=log(10)=1. We can substitute either 1-log(5)=log(2) or 1-log(2)=log(5). I'll go with the first choice: 120log(2)=120-120log(5), and then go from there.

    • @davidjames1684
      @davidjames1684 3 года назад

      @@justabunga1 My main point is that log approximations do not always work for problems like this, when both terms are very close, as they are in my example. I used 5^59 and 2^137 purposely to illustrate this. The difference between those is less than half of 1%. Also, I disagree with having to treat log(2) and log(5) as an exact answer only. In many (just not all) cases, the approximations of log(2) = 0.3 and log(5) = 0.7 are useful, especially when comparing 2 expressions that are fairly far apart in value (such as 5^51 and 2^120). My example is a counterexample where these approximations fail (lead you to believe that 5^59 is larger than 2^137, when actually 2^137 is slightly larger).

    • @justabunga1
      @justabunga1 3 года назад

      @@davidjames1684 we have to assume that log(2) and log(5) are unknown values to the approximation since both of them are not powers of 10, but I agree what you said so far. It's not like log(10)=1, log(100)=2, etc. and is between 0 and 1. Examples like pi, sqrt(2), or any irrational numbers are impossible to think this in your head without using your calculator. Anything that is an irrational number will always have to be left as an exact answer even though you can think this in your head as an approximate answer and can sometimes lead to different answers with the rounding and error, which is why I wouldn't do that. Something like larger numbers will have to use some other method that can be broken into some parts. Like you said 5^59 and 2^137 is impossible to think this in your head without the use of calculator, but we know is by the statement of equalities/inequalities: If log(a)>log(b), then a>b (in the same way that log(a)

    • @golddddus
      @golddddus 2 года назад

      The method used is universal. For your task, it was enough to use a logarithm with one decimal.
      For more accurate calculations, log (2) = 0.30103 ... log (5) = 1-0.30103 = 0.69897 log (2) is easy to remember. Draw one head. The left ear is 3, the left eye is 0, the nose is 1, the right eye is 0 and the right ear is 3. In my time of study, more than half a century ago, there were no computers. The memorized log (2) was useful to me in both chemical kinetics and radiochemistry.

  • @SakretteAmamiya
    @SakretteAmamiya 3 года назад +2

    51log5 = 51(0.6990) ~ 35
    120log2 = 120(0.3010) > 36
    Obviously, 2^120 is greater

    • @davidseed2939
      @davidseed2939 3 года назад

      i did it this way. quick and definite

    • @SakretteAmamiya
      @SakretteAmamiya 3 года назад

      @@davidseed2939 some basic logarithm values are really convenient for estimating, but not many people know this lol
      I only memorized logarithm values for 2, 3, 7 though, but they're quite enough for most cases

  • @raymondarata6549
    @raymondarata6549 Год назад

    Take log of both sides. Then use the power rule to get 51log(5) =? 120log(2). Rewrite to get log(5)/log(2) =? 120/51. Anyone with base 10 log experience knows that log(5) =.7 (.699 rounded off) and log(2) =.3 (.303 rounded off). .7/.3 = 7/3 = 2.33 and 120/51 is about 2.35. So, 5^51

  • @sujanshankarbhowmick9381
    @sujanshankarbhowmick9381 3 года назад

    For the summation 2+3^3+6+7^7+10+11^11+.............+(4x+2)+(4x+3)^(4x+3)=z^n, where n>1, how many +ve integer solutions for z are possible?

  • @mahajankeshav14
    @mahajankeshav14 2 года назад +1

    Very very good

  • @kathanshah8305
    @kathanshah8305 3 года назад +2

    Or just change 2 to 4 and then log it
    So it would be 60log4 and 51 log5

  • @divyamkothari4387
    @divyamkothari4387 3 года назад +1

    Both the values lie in x if x^0 -0^x = 1 over and out 🙂

  • @user-vr3jh9nv7b
    @user-vr3jh9nv7b 3 года назад +1

    なんとか粘って考え,
    5^51 , 2^120 = √5^102 , √2^240
    = (√5^3)^34 , (√2^7)^34.2857…
    = √125^34 < √128^34.2857…
    となりました。
    高校入試 数学 でもいけますね。

  • @orangeinternetcafe5381
    @orangeinternetcafe5381 2 года назад

    take log of 2 with base 5 and you will see that it comes out as 2¹¹⁸ which is less than 2¹²⁰ so b is greater

  • @rssl5500
    @rssl5500 3 года назад

    If both numbers are bigger than e then the number with base closer to e is bigger e is approximately 2.718 so 2 is obviously close hence 2^120 is bigger this is solved with differentiations on a function I think I’m not sure tho but a youtuber called momeme made a video about it

  • @uberless1
    @uberless1 3 года назад

    1:38
    Technically it *would* help if you could show that 5

  • @alessandrorossi1294
    @alessandrorossi1294 3 года назад

    Nice, I solved it the same way

  • @user-kh2fy3es1b
    @user-kh2fy3es1b Год назад

    Compare log to log:51*log(5) versus 2*log(120). Easy calculation by excel

  • @nimishnair9316
    @nimishnair9316 2 года назад

    Another way to do it was to take the natural log of 5^51 and 2^120 and then compared the quantities. 51•ln(5)= 82.1 and 120•ln(2)= 83.2 and clearly 2^120 is greater. But I understand your proof was for people who don't have a mathematical background or studied it a long time ago.

    • @SyberMath
      @SyberMath  2 года назад

      This eliminates the need for the memorization of the values of log2 and/or log5 or the use of calculators

  • @gustavoteixeira3103
    @gustavoteixeira3103 3 года назад

    I memorized log(2) = 0.30 (approximately, of course). Then log(5) = 1-log(2) = 0.7
    So 5 = 2^(0.7/0.3) = 5^(7/3)
    5^51 = 2^(7*51/3) = 2^119
    2^119

    • @justabunga1
      @justabunga1 3 года назад

      Let's not think about what the approximation of log(2) and log(5) just because you can think in your head. Just leave those as an exact answer. What can you tell about log(2) and log(5) if there is something in common? The answer has to do with the LCM, which is 10. That means log(2)+log(5)=log(10)=1. Rearrange this and make that as a substitution as either 1-log(2)=log(5) or 1-log(2)=log(2) and then go from there.

  • @vuyyurisatyasrinivasarao3140
    @vuyyurisatyasrinivasarao3140 3 года назад

    Good solution super logic

  • @einsteingonzalez4336
    @einsteingonzalez4336 3 года назад

    Can I use logarithms?

  • @MathElite
    @MathElite 3 года назад +2

    this is a joke btw.....
    5^51-2^120 = -8.8513879 x 10^35 so clearly.....2^120 is bigger

  • @avhuf
    @avhuf 3 года назад

    I'd be highly surprised if a 120th power of a number were to be smaller than a 51th power of a slightly bigger number.

  • @Noobmaster-pq5eu
    @Noobmaster-pq5eu 3 года назад +1

    Well my solution is:
    5^3 is less than 2^7
    (5^3)^17 is less than (2^7)^17
    5^51 is less than 2^119
    5^51 is less than 2^120

  • @sie_khoentjoeng4886
    @sie_khoentjoeng4886 3 года назад

    Using logatlritm:
    5^51 ~ 2^120
    Log (5^51) ~ Log(2^120)
    51 log 5 ~ 120 log 2
    51 log 5 / log 2 ~ 120
    Since log 4/log 2 < log 5/log 2 < log 8/log 2, means that 2

    • @robertveith6383
      @robertveith6383 3 года назад

      No, you cannot guess/estimate 2.3 for log(5)/log(2). What if the guess is 2.36? Then 51*2.36 =
      120.36 > 120. You are going to have to fix your proof.

    • @justabunga1
      @justabunga1 3 года назад

      @@robertveith6383 exactly. Although, I would not try guessing what the approximate answer of log(2) and log(5) is. Just go with the one where 2 and 5 have the LCM, which is 10. So, subsitute log(2)+log(5)=log(10)=1 as either 1-log(5)=log(2) or 1-log(2)=log(5) and then go from there.

  • @chandanamgoth2387
    @chandanamgoth2387 3 года назад

    Just take log on both sides and powers will come down and you can calculate in like a minute

  • @tubeofvideos
    @tubeofvideos 2 года назад

    You can use log function very easily.

  • @ziyabayramov2188
    @ziyabayramov2188 2 года назад

    I solved like -> 5^51 = (4 x 1.25)^51 = 2^102 x (1.25)^51 = 2^102 x (1.25^3)^17 = 2^102 x (1.953..)^17 < 2^102 x 2 ^ 18 = 2^120

  • @user-ei6rd7ei7x
    @user-ei6rd7ei7x 3 года назад

    notice that 2^7=128>125=5^3, hence 2^120>2^119=(2^7)^17>(5^3)^17=5^51

  • @puikihung5882
    @puikihung5882 3 года назад

    my suggestion is to compare 5(5^50) and 16^50

  • @VSN1001
    @VSN1001 3 года назад

    Is using AM => GM inequality possible?

  • @GDPlainA
    @GDPlainA 3 года назад

    all i did was to make 5^51 as 5(5^50) and then do like the beginning with 5(3125^10) < 4096^10. by observation, 4096^10 will be significantly greater than 3125^10 so multiplying 5 to 3125^10 will not make it greater than 4096^10
    Therefore, 5^51 is less than 2^120

  • @Viewer2812
    @Viewer2812 3 года назад

    I managed to do this one on my own:
    5^51 vs 2^120
    5^3=125
    2^7=128
    5^51 vs 2^120
    (5^3)^17 vs (2^7)^17 x2
    (125)^17 vs 2(128)^17
    128>125
    128^17>125^17
    2(128)^17>125^17
    Therefore 2^120 is greater. This is quite fun.

  • @shhi9379
    @shhi9379 2 года назад

    Please compare 2^223 and 5^96

  • @seroujghazarian6343
    @seroujghazarian6343 3 года назад +1

    5^1 and 2^2 are closer to each other than 5^3 and 2^7...

  • @sachinmaheswar3028
    @sachinmaheswar3028 3 года назад

    2pow2.32 is almost 5. So I can write 2pow(2.32*51) which will be almost 2pow118. So 2pow120 will be greater.

  • @sn4592
    @sn4592 2 года назад

    Take nature log on both sides 51ln5

  • @rajatchopra5829
    @rajatchopra5829 3 года назад

    Using logs,
    2^120=5^(51.68) which is greater than 5^51

  • @devondevon3416
    @devondevon3416 3 года назад

    2^120 is greater Answer 2^120 (6:59)
    2^120 vs 5^51
    5^51 can be written as 5^3^17 or 125^17 (3x17=51) o
    2^120 can be written as 2^7^17.14 (since 7 x 17.14=120), but 2^7= 128, so 2^120= 128^17.14
    128^17.14 is greater than 125^ 17 Therefore 2^120 is greater since it is equivalent to 128^17.14
    PS thought I did it differently from you but realize that not that much as it was the same approach such as looking for a 2^n and 5^m which is close (125 and 128). The only difference was that you reduced 120 to 119 since both 57 and 119 are divisible by 17, and hoped to prove that if
    5^51 (or 125^17) was less than 2^119 then it was less than 2^120. I, instead, used 120 and divided it by 7
    to get 17.14 which is greater than 17 (yet, luckily, very very close to 17).

  • @brianneill4376
    @brianneill4376 2 года назад

    So, 5+511×2+120>0^3=:"120 Zeros with a 2 after it is larger than 5 with 51 zeros after it.

  • @user-kj7hr3qw9w
    @user-kj7hr3qw9w 3 года назад

    أنت مبدع

  • @luccavelier9514
    @luccavelier9514 3 года назад

    Multiply both sides by 2^51
    You compare 10^51 and 2^171
    2^10 = 1024 > 10^3
    2^171 > (10^3)^17,1 = 10^51,3 > 10^51

  • @ivankaznacheyeu4798
    @ivankaznacheyeu4798 3 года назад

    Ive solved it so
    5^51 2^120
    5^17 2^40
    2^10 > 10^3 then 2^40 > 10^12 = 2^12 * 5^12 > 5^5 * 5^12 = 5^17 then 2^120 > 5^51
    But solution with 125 128 pair is much better

  • @davidjames1684
    @davidjames1684 3 года назад

    Try 5^59 vs. 2^137.

  • @leoofficial527
    @leoofficial527 2 года назад

    The greater the power, the higher the chance to be greater

  • @emailid7862
    @emailid7862 3 года назад

    He's using notability!

  • @mrbenwong86
    @mrbenwong86 3 года назад

    2^120 is a easy, we also know is smaller than 2^119, but is it smaller than 2^118?

    • @ignaciorodriguez639
      @ignaciorodriguez639 3 года назад +1

      Notice that
      ( 128 / 125 ) ^ 17 = 2 ^ 119 / 5 ^ 51
      Look for powers of 128 / 125
      128 / 125 = 1.024
      ( 128 / 125 ) ^ 2 = 1.024 ^ 2 < 1.05
      ( 128 / 125 ) ^ 6 < 1.05 ^ 3 < 1.2
      ( 128 / 125 ) ^ 18 < 1.2 ^ 3 < 1.8
      Therefore
      ( 128 / 125 ) ^ 17 < 2
      Which implies
      2 ^ 118 < 5 ^ 51